Tải bản đầy đủ (.pdf) (27 trang)

Một số bất đẳng thức hình học trong tam giác

Bạn đang xem bản rút gọn của tài liệu. Xem và tải ngay bản đầy đủ của tài liệu tại đây (263.85 KB, 27 trang )

Hội thảo Khoa học, Sầm Sơn 28-28/09/2019

MỘT SỐ BẤT ĐẲNG THỨC HÌNH HỌC
TRONG TAM GIÁC
Nguyễn Văn Ngọc
Trường Đại học Thăng Long

Tóm tắt nội dung

1

Mở đầu

Bất đẳng thức hình học nói chung và vất đẳng thức trong tam giác nói riêng là chun
mục khó trong lĩnh vực Tốn Phổ thơng, nhưng lại có sức hấp dẫn kỳ lạ, bởi vì các bất
đẳng thức này khơng chỉ có ý nghĩa về nội dung mà cịn khá đẹp về hình thức và địi hỏi
nhiều sáng tạo.
Bất đẳng thức trong tam giác là chuyên mục lý thú, hấp dẫn nhiều thế hệ những
chuyên gia toán học, những người dạy và người học toán trong các trường cấp Trung
học ở khắp nới trên Thế giới.
Hầu hết các bất đẳng thức trong tam giác hiện nay có trong các tài liệu chun khảo
đã được tìm ra từ nhiều thế kỷ trước, nhất là Thế kỷ 20. Các bất đẳng thức này có thể tìm
thấy trong nhiều tài liệu, thí dụ như các tài liệu [1] và [5].
Nhiều bất đẳng thức trong tam giác mới được tìm ra trong khoảng vài thập niên gần
đây (xem, [3, 4, 7,6] và các tài liệu trong đó), nhưng chưa được hệ thống và giới thiệu ở
Việt Nam. Mục đích của bài viết này là trình bày một số các bất đẳng thức kinh điển và
bất đẳng thức mới trong tam giác, giúp người dạy và học Toán ở bậc phổ thơng có thêm
tư liệu phục vụ cho cơng việc giảng dạy và học tập.

2


Bổ trợ

2.1

Các bất đẳng thức kinh điển của dãy số

Chúng ta sẽ cần đến các bất đẳng thức quan trọng sau đây
Định lý 2.1. (Bất đẳng thức AM − GM).Với n số thực khơng âm bất kì a1 , a2 , . . . , an , ta có bất
đẳng thức

a1 + a2 + . . . + a n
n
a1 .a2 . . . . .an
n
Đẳng thức xảy ra khi và chỉ khi a1 = a2 = . . . = an

1


Hội thảo Khoa học, Sầm Sơn 28-28/09/2019
Định lý 2.2 (Bất đẳng thức lũy thừa). Với mọi bộ số không âm a1 , a2 , . . . , an và m = 1, 2, . . .
ta đều có
a1m + a2m + . . . + am
a1 + a2 + · · · + a n m
n
n
n
Định lý 2.3 (Bất đẳng thức Cauchy - Schawrz). Xét hai bộ số thực tùy ý a1 , a2 , · · · , an và
b1 , b2 , · · · , bn . Khi đó ta có


( a1 b1 + a2 b2 + · · · + an bn )2

( a21 + a22 + · · · + a2n )(b12 + b22 + · · · + bn2 ),

a2
an
a1
=
= ··· = .
b1
b2
bn
(Với quy ước nếu mẫu bằng 0 thì tử cũng bằng 0).
Đẳng thức xảy ra khi và chỉ khi

Định lý 2.4 (Minkovskii). Với hai bộ số thực { ak }nk=1 , {bk }nk=1 có bất đẳng thức
n



n

a2k + bk2 ≤

k =1

2.2

∑ ak


2

n

+

k =1

∑ bk

2

.

k =1

Các đại lượng và định lý thông dụng của một tam giác

Trong phần này ta luôn giả sử tam giác ABC có:
• BC = a, CA = b, AB = c;
• S là diện tích tam giác;
• p là nửa chu vi tam giác;
• m a , mb , mc , wa , wb , wc , h a , hb , hc lần lượt là độ dài các trung tuyến, các phân giác và
các đường cao tương ứng với các cạnh a, b, c;
• r, R, r a , rb , rc lần lượt là các bán kính đường trịn nội tiếp, đường trịn ngoại tiếp,
đường tròn bàng tiếp với các cạnh a, b, c của tam giác ABC.
• ∑ a = a + b + c.
Định lý 2.5 (Định lý hàm số sin). Trong mọi tam giác ABC có hệ thức
a
b

a
=
=
= 2R.
sin A
sin B
sin A
Định lý 2.6 (Định lý hàm số cosin). Trong mọi tam giác ABC có hệ thức
a2 = b2 + c2 − 2bc cos A,
b2 = c2 + a2 − 2ca cos B,
c2 = a2 + b2 − 2ab cos C.

Như một hệ quả của Định lý hàm số cosin, ta có khẳng định Địnhnlý Pitago nổi tiếng
Định lý 2.7 (Định lý Pitago). Tam giác ABC vuông tại A khi và chỉ khi
a2 = b2 + c2 .

2


Hội thảo Khoa học, Sầm Sơn 28-28/09/2019
Định lý 2.8 (Định lý Apollonius-Pappus). Tam giác ABC có các hệ thức sau đây về đường
trung tuyến
2b2 + 2c2 − a2
,
4
2c2 + 2a2 − b2
m2b =
,
4
2a2 + 2b2 − c2

m2c =
.
4
m2a =

Định lý 2.9 (Định lý đường phân giác). Tam giác ABC có các hệ thức sau đây về các đường
phân giác
wa =

2ca
2ab
2bc
, wb =
, wc =
.
b+c
c+a
a+b

Định lý 2.10 (Về diện tích của tam giác). Diện tích S của tam giác ABC được tính theo các
cơng thức
1
1
1
ah a = bhb = chc
2
2
2
1
1

1
= ab sin C = bc sin A = ca sin B
2
2
2
abc
=
4R
= sr

S=

= ( p − a )r a = ( p − b )r b = ( p − c )r c
=

p( p − a)( p − b)( p − c), ( Cơng thức Heron ).

Định lý 2.11 (Bán kính đường trịn nội tiếp). Trong tam giác ABC ta có
r = ( p − a) tan

B
C
A
= ( p − b) tan = ( p − c) tan .
2
2
2

Định lý 2.12 (Bán kính đường trịn bàng tiếp). Trong tam giác ABC ta có
r a = p tan


A
B
C
, rb = p tan , rc = p tan .
2
2
2

Định lý 2.13 (Công thức Euler). Trong tam giác ABC ta có
R( R − 2r ) = OI 2 ,
trong đó O và I tương ứng ký là tâm của đường tròn ngoại tiếp và tâm của đường tròn nội tiếp
tam giác ABC.

3


Hội thảo Khoa học, Sầm Sơn 28-28/09/2019

3 Một số bất đẳng thức kinh điển liên quan đến
đường cao và đường trung tuyến
Mệnh đề 3.1. Trong mọi tam giác có bất dẳng thức
9r ≤ h a + hb + hc ≤

9
R.
2

(3.1)


Chứng minh. Từ cơng thức của diện tích ram giác ta có
ha =

2sr
2sr
2sr
, hb =
, hc =
.
a
b
c

Suy ra
h a + hb + hc = r ( a + b + c)

1 1 1
+ +
a b
c

≥ 9r.

Do đó vế trái của (3.1) được chứng minh. Cũng từ cơng thức diện tích của tam giác ta có
ha =

ac
ab
bc
, hb =

, , hc =
.
2R
2R
2R

Suy ra
h a + hb + hc =

a2 + b2 + c2
bc + ca + ab

.
2R
2R

Theo Định lý hàm số sin ta có
9
a2 + b2 + c2
= 2R(sin2 A + sin2 B + sin2 C ) ≤ R.
2R
2
Vậy vế phải của (3.1) cũng được chứng minh. Mệnh đề được chứng minh.
Mệnh đề 3.2. Trong mọi tam giác ABC có bất dẳng thức

3
h a + hb + hc ≤
( a + b + c ).
2


(3.2)

Dấu đẳng thức xảy ra khi và chỉ khi tam giác ABC là đều.
Chứng minh. Ta có bất đẳng thúc

( a + b + c)2 ≥ 3(bc + ca + ab) =

3abc
(h a + hb + hc ) = 6R(h a + hb + hc ).
2S

Mặt khác lại có

( a + b + c )2
1
≤ ( a2 + b2 + c2 ) = 8R2 (1 + cos A cos B cos C ) = 8R2 1 +
= 9R2 .
3
8
Suy ra
a+b+c ≤



6
3R, ( a + b + c)2 ≥ √ ( a + b + c)(h a + hb + hc ).
3 3

Từ đây ta có điều phải chứng minh.


4


Hội thảo Khoa học, Sầm Sơn 28-28/09/2019
Mệnh đề 3.3. Trong mọi tam giác ABC có bất đẳng thức
m a + mb + mc ≤ 4R + r.

(3.3)

Dấu đẳng thức xảy ra khi và chỉ khi tam giác ABC là đều.
Chứng minh. Ký hiệu p a , pb , pc tương ứng là khoảng cách từ tâm vòng tròn ngoại tiếp
tam giác ABC đến các cạnh BC, CA, AB. Khi đó
m a ≤ R + p a , mb ≤ R + pb , mc ≤ R + pc .
Theo công thức tính diện tích của tam giác, ta có
ap a = R2 sin 2A, bpb = R2 sin 2B, cpc = R2 sin 2C.
Suy ra
p a + pb + pc = R(cos A + cos B + cos C ) = R 1 +
Vậy ta có
m a + mb + mc ≤ 3R + R 1 +

r
R

r
.
R

= 4R + r.

Mệnh đề được chứng minh.

Mệnh đề 3.4. Trong mọi tam giác ABC có bất đẳng thức

m2a + m2b + m2c ≥ 3 3S.

(3.4)

Dấu đẳng thức xảy ra khi và chỉ khi tam giác ABC là đều.
Chứng minh. Ta có
m2a + m2b + m2c =

3 2
( a + b2 + c2 ).
4

Chúng ta chứng minh bất đẳng thức Weisenbock


a2 + b2 + c2 ≥ 4 3S.
Thật vậy, ta có


√ 1
a2 + b2 + c2 − 4 3S = 2(b2 + c2 ) − 2bc cos A − 4 3 sin A
2

3
1
π
2
2

= 2(b + c ) − 4bc cos A +
sin A = 2(b2 + c2 ) − 4bc cos( A − )
2
2
3
≥ 2(b2 + c2 ) − 4bc = 2(b − c)2 ≥ 0.
π
Dấu đẳng thức xảy ra khi và chỉ khi b = c và A = , tức tam giác ABC đều. Vậy ta có
3
m2a + m2b + m2c =


3 2
3 √
( a + b2 + c2 ) ≥ 4. 3S = 3 3S.
4
4

Mệnh đề được chứng minh.

5


Hội thảo Khoa học, Sầm Sơn 28-28/09/2019

4 Bất đẳng thức Brown-Barrero về đường trung
tuyến
Mục này trình bày một bất đẳng thức về đường trung tuyến do Brown p.H. và DiazBarrero J.L. nhận được trong [2]. Trước hết ta cần Bổ đề sau đây.
Bổ đề 4.1. Cho ABC là tam giác có độ dài các cạnh là a, b, c và bán kính đường trịn nội tiếp là
r. Khi đó

a2 ( b + c )
abc
(4.1)
∑ (a + b)(a + c) ≤ 8r2 .
quay vòng
Chứng minh. Để chứng minh bất đẳng thức trên, chúng ta thêm số hạng p =
và sẽ chỉ ra



quay vịng
Từ cơng thức đã biết S =

a+b+c
2

a2 ( b + c )
a+b+c
abc

≤ 2.
( a + b)( a + c)
2
8r

(4.2)

abc
= pr kết hợp với bất đẳng thức Euler R ≥ 2r, ta có
4R

p=

abc
abc
≤ 2
4Rr
8r

và vế phải của bất đẳng thức được chứng minh. Để chứng minh vế trái của bất đẳng
thức, ta chú ý rằng
2a2 (b + c)
(4.3)
∑ (a + b)(a + c) ≤ a + b + c.
quay vòng
Thật vậy, ta có



quay vịng

4( a2 b2 + b2 c2 + c2 a2 + a2 bc + ab2 c + abc2 )
a2 ( b + c )
=
( a + b)( a + c)
( a + b)(b + c)(c + a)

( a + b + c)( a + b)(b + c)(c + a) − ab( a − b)2 − bc(b − c)2 − ca(c − a)2
( a + b)(b + c)(c + a)
2
ab( a − b) + bc(b − c)2 + ca(c − a)2

= ( a + b + c) −
≤ a+b+c
( a + b)(b + c)(c + a)
=

như mong muốn. Dấu bằng xảy ra nếu a = b = c. Tức là, khi tam giác là tam giác đều.
Bổ đề được chứng minh.
Định lý 4.1. Cho ABC là tam giác có độ dài các trung tuyến ứng với các cạnh là m a , mb b, mc và
diện tích S. Khi đó



quay vịng

m5 + m5b + m5c
m2a (mb + mc )
≤ a
.
(m a + mb )(m a + mc )
6S2

Chứng minh. Trong bất đẳng thức (4.1) ta đặt
a=

2
2
2
m a , b = mb , c = mc ,
3
3

3

6

(4.4)


Hội thảo Khoa học, Sầm Sơn 28-28/09/2019
sử dụng công thức
r=

S
m a + mb + mc

và bất đẳng thức trung bình AM-GM, ta có



quay vịng
1
≤ 2
2S

m2a (mb + mc )
( m a + m b + m c )2 ( m a m b m c )

(m a + mb )(m a + mc )
18S2

m a + mb + mc

3

2

m a + mb + mc
3

Dấu bằng xảy ra khi m a = mb = mc . Tức là, khi
chứng minh.

5
5.1

3



m5a + m5b + m5c
.
6S2

ABC là tam giác đều, suy ra điều phải

Bất đẳng H.Y.Yin mở rộng
Giới thiệu

Mục này trình bày các kết quả của Jian Liu [4] chứng minh một bất đẳng thức đã biết
liên quan đến cận trên của tổng đường trung tuyến là mạnh. Tác giả cũng đã chứng minh
một giả thiết mạnh hơn về bất đẳng thức này, tương đương với bất đẳng thức được đề
xuất bởi H.Y.Yin trong [1].

Cho ABC là tam giác có các đường trung tuyến m a , mb , mc và nửa chu vi p, khi đó ta
có bất đẳng thức sau (xem [1]):
m a + mb + mc < 2p,

(5.1)

trong đó hằng số 2 là tối ưu.
Năm 2000, Chu Xiao-Guang và Yang Xue-Zhi (xem tài liệu trong [4]) thiết lập một bất
đẳng thức mới mạnh hơn, được cho bởi Định lý sau đây.
Định lý 5.1. Trong tam giác ABC bất kỳ với các đường trung tuyến m a , mb , mc , nửa chu vi p,
bán kính đường trịn nội tiếp r và bán kính đường trịn ngoại tiếp R, bất đẳng thức sau đúng:

(m a + mb + mc )2 ≤ 4p2 − 16Rr + 5r2 .

(5.2)

Dấu bằng xảy ra khi và chỉ khi tam giác ABC là tam giác đều.
Bất đẳng thức (5.2) là một cận trên xuất sắc cho biểu diễn (m a + mb + mc )2 . Nhưng
các tác giả trên vẫn chưa trả lời một câu hỏi tự nhiên là: tìm giá trị cực đại λ sao cho bất
đẳng thức
(m a + mb + mc )2 ≤ 4p2 − λRr + (2λ − 27)r2 .
(5.3)
đúng với mọi tam giác ABC.
Một trong các mục tiêu của bài báo này là chứng minh kết luận liên quan sau:
Định lý 5.2. Giả sử λ là số thực dương sao cho bất đẳng thức (5.3) đúng với mọi tam giác ABC.
Khi đó λmax = 16.
Bất đẳng thức (5.3) trở thành (5.2) khi k = 16. Điều này có nghĩa bất đẳng thức (5.2)
là mạnh nhất trong tất cả các bất đẳng thức dạng (5.3).

7



Hội thảo Khoa học, Sầm Sơn 28-28/09/2019

5.2

Chứng minh của Định lý 5.2

Chứng minh. Nếu R = 2r, thì bất đẳng thức (5.3) tương đương với
λ≤

4p2 − 27r2 − (m a + mb + mc )2
.
r ( R − 2r )

Theo các đẳng thức đã biết pr2 = ( p − a)( p − b)( p − c), abc = 4Rr, ta thấy thêm rằng bất
đẳng thức trên tương đương với
λ≤

4p3 − 27( p − a)( p − b)( p − c) − p(ma + mb + mc)2
.
abc − 8( p − a)( p − b)( p − c)

(5.4)

Bây giờ ta giả sử ABC là tam giác cân với các cạnh x, 1, 1 ( x = 1), đặt b = c = 1, a = x,
1√ 2
rồi dùng công thức trung tuyến m a =
2b + 2c2 − a2 và các bất đẳng thức đã biết:
2

p=

abc

1
( a + b + c ), R =
2
4

( p − a)( p − b)( p − c)

,r =

( p − a)( p − b)( p − c)
,
p

ta được
1
1
1
4 − x2 , mb = mc =
x + 1, m a =
2
2√
2
1
x 4 − x2
R= √
,r =

.
2( x + 2)
4 − x2
p=

1 + 2x2

Thay a = x, b = c = 1 và các hệ thức trên vào (5.4), sau một vài tính tốn ta được
λ≤

12x3 − 22x2 + 20x − 2( x + 2) (4 − x2 )(1 + 2x2 ) + 8
.
x ( x − 1)2

Đặt
f ( x ) = 12x3 − 22x2 + 20x − 2( x + 2)

(4 − x2 )(1 + 2x2 ) + 8, g( x ) = x ( x − 1)2 .

Thì suy ra
f (x) =

12x4 + 16x3 − 28x2 − 28x − 8 + 4(9x2 − 11x + 5)

(4 − x2 )(1 + 2x2 )

(4 − x2 )(1 + 2x2 )

g ( x ) = 3x2 − 4x + 1.
Lưu ý rằng

lim f ( x ) = 0, lim g( x ) = 0.

x →0

x →0

Theo bất đẳng thức (5.5) và quy tắc L’Hopital, ta được
λ ≤ lim

x →0

= lim

x →0

f (x)
f (x)
= lim
x →0 g ( x )
g( x )
12x4 + 16x3 − 28x2 − 28x − 8 + 4(9x2 − 11x + 5)

(3x2 − 4x + 1)

= 16.
Do đó ta chứng minh xong định lý 5.2.

8

(4 − x2 )(1 + 2x2 )

(4 − x2 )(1 + 2x2 )

(5.5)


Hội thảo Khoa học, Sầm Sơn 28-28/09/2019
Nhận xét 5.1. Chu Xiao-Guang và Yang Xue-Zhi (xem tài liệu trong [4]) cũng thu được
bất đẳng thức ngược của (5.2):

(m a + mb + mc )2 ≥ 4p − 28Rr + 29r2 .

(5.6)

Theo cách tương tự như chứng minh của Định lý 5.2, ta cũng chứng minh được kết luận
sau: Cho λ là số thực dương thỏa mãn

(m a + mb + mc )2 ≥ 4p − λRr + (2λ − 27)r2 .

(5.7)

với mọi tam giác ABC, khi đó λmin = 28.

6

Bất đẳng thức mới về tổng các đường trung tuyến

Trong mục này, chúng tôi giới thiệu một bất đẳng thức mới về tổng của các đường
trung tuyến trong tam giác và hai bất đẳng bất đẳng thức khác được chứng minh bằng
cách sử dụng bất đẳng thức này [3].
Định lý 6.1. Trong mọi tam giác ABC với các cạnh a, b, c, đường trung tuyến m a , mb , mc , bán

kính đường trịn nội tiếp r, bán kính đường trịn ngoại tiếp R, bất đẳng thức sau đúng:

( m a + m b + m c )2
r2
≤ 2 + 2,
2
2
2
a +b +c
R
với dấu bằng xảy ra khi và chỉ khi

(6.1)

ABC là tam giác đều.

Gần đây, bất đẳng thức (6.1) đã được chứng minh bởi Liu J.(xem, [3, 4]). Tuy nhiên,
chứng minh này rất phức tạp. Tác giả dùng một số bổ đề , trong đó có bất đẳng thức
4mb mc ≤ 2a2 + bc −

4p( p − a)(b − c)2
,
2a2 + bc

(6.2)

với dấu bằng xảy ra khi và chỉ khi b = c.
Chứng minh của Định lý 6.1 khá dài, nên chúng tơi khơng trình bày ở đây.
Trong mục này, ta sẽ áp dụng Định lý 6.1 để thiết lập hai bất đẳng thức mới, mà vẫn
chưa được chứng minh bằng cách dùng bất đẳng thức (5.2).

Định lý 6.2. Với mọi tam giác ABC ta có:
m a + mb + mc − (h a + hb + hc ) ≤ 2( R − 2r ),
với dấu bằng xảy ra khi và chỉ khi

(6.3)

ABC là tam giác đều.

Chứng minh. Theo Định lý 6.1, để chứng minh (6.3) ta cần chứng minh

( a2 + b2 + c2 ) 2 +

r2
R2

≤ [h a + hb + hc + 2( R − 2r )]2 .

(6.4)

Nhân cả hai vế với 4R2 và dùng hệ thức 2Rh1 = bc, v.v., bất đẳng thức (6.4) trở thành
dạng tương đương sau:
M0 ≡ [bc + ca + ab + 4R( R − 2r )]2 − 4( a2 + b2 + c2 )(2R2 + r2 ) ≥ 0.

9

(6.5)


Hội thảo Khoa học, Sầm Sơn 28-28/09/2019
Áp dụng đẳng thức [5]


∑ a2 = 2p2 − 8Rs − 2r2

và đẳng thức đã biết
bc + ca + ab = p2 + 4Rr + r2 ,

(6.6)

dễ dàng thu được
M0 = (4R2 + 4Rr + 3r2 − p2 )2 .
Từ đó rút ra bất đẳng thức M0 ≥ 0 và (6.3) được chứng minh. Rõ ràng dấu bằng trong
(6.3) chỉ xảy ra khi ABC là tam giác đều. Định lý được chứng minh.
Nhận xét 6.1. Bằng phương pháp chứng minh Định lý 2 trong [4], ta có thể chứng minh
rằng hằng số 2 ở vế phải của (6.3) là tối ưu. Thêm vào đó, từ bất đẳng thức Leuenberger
[1]:
h a + hb + hc ≤ 2R + 5r,
(6.7)
ta thấy rằng bất đẳng thức (6.3) mạnh hơn kết quả sau:
m a + mb + mc ≤ 4R + r.

(6.8)

Nhận xét 6.2. Bằng cách sử dụng bất đẳng thức (5.2), ta dễ dàng chứng minh bất đẳng
thức tuyến tính cho tổng m a + mb + mc :

m a + mb + mc ≤ 2p − (6 3 − 9)r.
(6.9)
Bất đẳng thức này cũng mạnh hơn (6.8) vì ta có bất đẳng thức sau:

p ≤ 2R + (3 3 − 4)r


(6.10)

của W.J. Blundon (xem tài liệu trong [3]).
Tiếp theo, ta chứng minh một bất đẳng thức về tam giác nhọn, mà được tác giả tìm ra
trong nhiều năm trước nhưng vẫn chưa chứng minh được cho tới bây giờ.
Định lý 6.3. Cho tam giác nhọn ABC ta có:
h a + hb + hc
1
r
≥ + .
m a + mb + mc
2 R

(6.11)

Dấu bằng xảy ra khi và chỉ khi tam giác nhọn ABC là tam giác đều.
Chứng minh. Theo Định lý 6.1, để chứng minh (6.11), ta chỉ cần chứng minh rằng
2

( h a + hb + hc ) −

1
r
+
2 R

2

2+


r2
R2

( a2 + b2 + c2 ) ≥ 0.

(6.12)

Nhân cả hai vế bất đảng thức trên rồi dùng hệ thức 2Rh a = bc, v.v., ta thấy (6.12) tương
đương với
N0 ≡ R2 (bc + ca + ab)2 − ( R + 2r )2 (2R2 + r2 )( a2 + b2 + c2 ) ≥ 0.

(6.13)

Thay (??) và (6.6) vào biểu thức của N0 , thì (6.13) tương đương với
N0 ≡ p4 R2 − 4( R4 + 2R3 r + 4R2 r2 + 2Rr3 + 2r4 ) p2

+ (4R + r )(4R4 + 20R3 r + 19R2 r2 + 8Rr3 + 8r4 ) ≥ 0.

10

(6.14)


Hội thảo Khoa học, Sầm Sơn 28-28/09/2019
Ta viết lại N0 như sau
N0 = 4r2 ( R + 2r )(2R2 + r2 )e + 8r4 G2 + R[4r (3R + r )eRG1 ]C0 ,

(6.15)


trong đó
e = R − 2r
G1 = p2 − 16Rr + 5r2
G2 = 4R2 + 4Rr + 3r2 − p2
C0 p2 − (2R + r )2 .
Do đó theo bất đẳng thức Euler e ≥ 0, bất đẳng thức Gerretsen G1 ≥ 0, G2 ≥ 0, và bất
đẳng thức tam giác nhọn C0 ≥ 0 của Ciamberlini, ta kết luận rằng N0 ≥ 0 đúng cho tam
giác nhọn ABC. Nên bất đẳng thức (6.12) và (6.11) được chứng minh. Dễ kiểm tra được
dấu bằng trong (6.11) xảy ra khi ABC là tam giác đều. Đinhj lý được chứng minh.

7 Bất đẳng thức mới về tổ hợp tuyến tính của các
đường cao và đường trung tuyến
Mục này trình bày một số bất đẳng thức về tổ hợp tuyến tính của các đường cao và
đường trung tuyến dựa trên các kết quả Zhivko Zhelev trong [6] năm 2008.
Hình học về tam giác là một lĩnh vực của hình học cở sở nơi các kết quả mới thú vị
luôn bật lên. Có rất nhiều định lý liên quan đến hình học về tam giác, bao gồm hàng trăm
các bất đẳng thức hình học (ví dụ xem [1]). Kết quả sau đây bao gồm hai bất đẳng thức
mà trông khá dễ chịu, nhưng bất đẳng thức thứ hai hóa ra là vơ cùng khó khăn để giải
quyết. Thực ra, chúng tơi chứng minh kết quả sau.
Định lý 7.1. Cho ABC là tam giác bất kỳ với các cạnh a, b và c. Đặt h a , hb , hc và m a , mb , mc
tương ứng là đường cao và trung tuyến của các cạnh. Khi đó hai bất đẳng thức sau đúng:



bch a + achb + abhc



am a + bmb + cmc ≤ bcm a + acmb + abmc .


ah a + bhb + chc ≤



(7.1)
(7.2)

Theo như những gì tác giả biết, bất đẳng thức thứ hai vẫn chưa được chứng minh.
Nhưng kiểm tra ngẫu nhiên các giá trị cạnh tam giác khơng tìm ra phản ví dụ.
Chứng minh của chúng tôi dựa trên lý thuyết hàm một biến và hai biến mà chúng tôi
cho rằng đây là một trở ngại vì bài tốn bên trên thuộc lĩnh vực tốn học sơ cấp.
Chứng minh của Định lý 7.1. • Để chứng minh bất đẳng thức (7.1) ta sử dụng ah a =
bhb = chc = 2S ABC . Khi đó



ah a + bhb + chc − bch a − achb − abhc



2S bc 2S ac 2S ab
= 6S −


a
b
c




bc
ac
ab
= 2S 3 −


.
a
b
c

11


Hội thảo Khoa học, Sầm Sơn 28-28/09/2019
Do đó,




ah a + bhb + chc ≤ bch a + achb + abhc



bc
ac
ab


≤0

⇔ 3−
a
b
c
3

3

3

⇔ 3abc ≤ (bc) 2 + (ca) 2 + ( ab) 2 , a > 0, b > 0, c > 0.



Thay x = bc, y = ac, z = ab, ta được
3

3

3

3abc ≤ (bc) 2 + (ca) 2 + ( ab) 2

⇔ 3xyz ≤ x3 + y3 + z3 , x > 0, y > 0, z > 0.
Bất đẳng cuối cùng hiển nhiên đúng theo bất đẳng thức Am-GM và ta chứng minh xong
(7.1).
• Bây giờ chúng ta sẽ chúng minh bất đẳng thức (7.2). Khơng giảm tính tổng quát, ta
có thể giả sử với ABC rằng a ≥ b ≥ c. Có ba trường hợp có thể:
1.


ABC là tam giác đều, tức là a = b = c.

2.

ABC là tam giác cân, tức là a = b > c.

3.

ABC là tam giác tùy ý, tức là a > b > c.

Trước tiên, ta viết lại (7.2) dưới dạng



( a − bc)m a + (b − ac)mb + (c − ab)mc ≤ 0.
(7.3)



Trường hợp thứ nhất. Nếu ABC là tam giác đều, thì a − bc = b − ac = c − ab = 0
và (7.3) là đẳng thức. Ở bên dưới ta sẽ thấy trong trường hợp này thật ra là trường hợp
cực trị của bài toán.
Trường hợp thứ hai. Bây giờ cho ABC là tam giác đều ta có thể giả sử a = b > c > 0.
Mặt khác ta có
1
2
1
mc =
2


ma =

1
2
1
2a2 + 2b2 − c2 =
2
2b2 + 2c2 − a2 =

a2 + 2c2 = mb ,
(7.4)
4a2

− c2 .

Sử dụng (7.4), (7.3) trở thành

(a −



ac)

a2 + 2c2 ≤

a−c
2

12


4a2 − c2 ,

a > c.

(7.5)


Hội thảo Khoa học, Sầm Sơn 28-28/09/2019
Ta sẽ chứng minh (7.5). Sau một số tính tốn, ta thu được:





a2 + 2c2 ≤

a−c
2

4a2 − c2 , a > c

√ √

√ √

( a − c)( a + c)
2
2
⇔ a( a − c) a + 2c ≤
2



3
2
2
2
⇔ 2 a + 2ac ≤ ( a + c) 4a − c

4( a3 + 2ac2 )
≤ a + c + 2 ac
2
2
4a − c

4a3 + 8ac2 − 4a3 + ac2 − 4a2 c + c3

2
ac
4a2 − c2

(a −

ac)

4a2 − c2

2

c3
4a2 − c2

2
9ac − 4a2 c

≤ 4ac

c6 + 81a2 c4 + 16a4 c2 + 18ac5 − 8a2 c4 − 72a3 c3 ≤ 64a5 c + 4ac5 − 32a3 c3

⇔ c6 + 14ac5 + 73a2 c4 − 40a3 c3 + 16a4 c2 − 64a5 c ≤ 0
⇔ c(c5 + 14ac4 + 73a2 c3 − 40a3 c2 + 16a4 c − 64a5 ) ≤ 0
(c5 + 14ac4 + 73a2 c3 − 40a3 c2 + 16a4 c − 64a5 ) ≤ 0
c 4
c 3
c 2
c
c 5
− 64 ≤ 0.
+ 14
+ 73
− 40
+ 16
a
a
a
a
a
c
Trong biểu diễn cuối cùng, đặt = t ∈ (0, 1), suy ra rằng
a
t5 + 14t4 + 73t3 − 40t2 + 16t − 64 ≤ 0


⇔ (t − 1) t4 + 15t3 + 88t2 + 48t + 64 ≤ 0.
<0

>0

nếu

t >0

Nhưng bất đẳng thức cuối cùng hiển nhiên đúng và chứng minh (7.5) trong trường hợp
này.
Trường hợp thứ ba. Bây giờ cho ABC là tam giác tùy ý và cho a > b > c > 0. Ta viết lại
(7.3) thành


1
1
( a − bc) 2b2 + 2c2 − a2 + (b − ca) 2a2 + 2c2 − b2
2
2
(7.6)

1
2
2
2
+ (c − ab) 2a + 2b − c ≤ 0
2
hay


1 2
a
1−
2

b
a

c
a

2

2

b
a

+2

c
a

2

−1+

b

a


c
a

2+2

c
a

2



b
a

2




F ( ba , ac ):= F ( x,y)

1
+ a2
2

c

a


b
a

2+2

b
a

2



c
a

2

≤ 0,

F ( ba , ac ):= F ( x,y)

(7.7)

13


Hội thảo Khoa học, Sầm Sơn 28-28/09/2019
1
và do vậy a2 F ( x, y) ≤ 0 ⇔ F ( x, y) ≤ 0.

2
Chúng tôi gọi hàm hai biến F ( x, y) định nghĩa như bên trên là hàm cá quỷ và bề mặt
đồ thị của hàm này trong R3 được gọi là mặt cá quỷ.
Theo ký hiệu mới bất đẳng thức (7.3) chuyển thành bài toán cực trị
max

( x,y)∈ M⊂R2

F ( x, t), trong đó
(7.8)

M = {( x, y) ∈ R2 | 0 ≤ y ≤ x ≤ 1, x + y ≥ 1},

trong đó M là tam giác một tam giác vuông và bất đẳng thức cuối cùng được suy ra từ
bất đẳng thức a + b > c trong tam giác bất kỳ.
Việc kiểm tra tính đúng đắn của hàm cá quỷ trên M là dễ dàng. Do đó, để chứng
minh (7.3), ta phải chứng minh max(x,y)∈ M F ( x, y) = 0.
Lưu ý rằng M là tập compact và vì M là hàm liên tục trên M, từ các kết quả thơng
thường trong giải tích thực suy ra rằng F ( x, y) đạt được cực đại ở đây. Để tìm điểm cực
đại này ta xét các điểm trong và điểm biên của M, M = intM ∪ ∂M.
Dùng các phần mềm tốn học, ví dụ như Maple ta thấy rằng

2(1 − xy) x
y 2x2 + 2y2 − 1
∂F ( x, y)
=−
+
+ 2 + 2y2 − x2

∂x

2 xy
2x2 + 2y2 − 1


( x − y) x
2 + 2x2 − y2
2( y − x ) x



,
+
2 + 2y2 − x2
2 x
2 + 2x2 − y2
và vì hàm cá quỷ là hàm đối xứng, tức là F ( x, y) = F (y, x ), ta suy ra

2(1 − xy)y
x 2y2 + 2x2 − 1
∂F ( x, y)
=−
+
+ 2 + 2x2 − y2

∂y
2 xy
2y2 + 2x2 − 1


2( x − y ) y

(y − x )y
2 + 2y2 − x2


+
.

2 + 2x2 − y2
2 y
2 + 2y2 − x2
Bây giờ ta giải hệ

∂F


=0
∂x
∂F


=0
∂y
ta được điểm tới hạn M1 (0, 9238127491 . . . , 0, 1660179102 . . .) và M2 (1, 1). Chú ý
M1 , M2 ∈ M và M1 ∈ intM, M2 ∈ ∂M.
Một lần nữa dùng phần mềm, ta được ma trận Hessian của hàm cá quỷ:
∂2 F ∂2 F
∂2 F
· 2−
2
∂x ∂y

∂x∂y


3 3
3 3

= −
2
2

2

H ( x, y) :=

| M2 (1,1)




3 3
3

4

=

9
4

2


≥0


∂2 F
3 3
và vì 2
=−
< 0, suy ra điểm M2 (1, 1) là điểm cực đại trên M. Để cho đầy
∂x | M2 (1,1)
2
đủ, ta cần kiểm tra điều gì xảy ra trên biên bởi vì M2 ∈ ∂M. Nhưng điều này là dễ dàng

14


Hội thảo Khoa học, Sầm Sơn 28-28/09/2019
và việc kiểm tra này bao gồm các trường hợp { x = 0}, { x = y}, { x = 1} và { x + y = 1}.
Điều này dẫn tới kết quả rằng điểm trong câu hỏi thật ra là điểm cực đại tuyệt đối trong
M. Điều này kéo theo
sup F ( x, y) = max F ( x, y) = F (1, 1) = 0.
( x,y)∈ M

( x,y)∈ M

Thêm vào đó, M1 là điểm cực tiểu địa phương và ngoài ra
F ( x, y)| M1 = F (0, 9238127491 . . . , 0, 1660179102 . . .) = −0, 4280657968 . . .
Mặt cá quỷ có thể nhìn thấy như sau:
Sử dụng phần mềm Maple hoặc Mathematica, ta có thể xem được các góc nhìn khác
nhau của mặt này:

Do đó tất cả các trường hợp của định lý được chứng minh.
Hệ quả sau là đúng.
Hệ quả 7.1. Cho a, b và c là các cạnh của tam giác ABC và cho m a , mb , mc tương ứng là
đường trung tuyến của các cạnh. Khi đó
a) (2p − 3a
3b)mb + (2p − 3c)mc ≥ 0.
√)m a +√(2p −√
ma
ab + ac + bc
b)
≤ 1, a ≥ b ≥ c.

mc
a+b+c

8 Một số bất đẳng thức kinh điển liên quan đến
đường phân giác
Chương này trình bày một số bất đẳng thức hình học kinh điển và các bất đẳng thức
mới liên quan đến đường phân giác và đường trung tuyến của tam giác. Nội dung cửa
chương này được hình thành chủ yếu từ các tài liêu [1, 7].
Mệnh đề 8.1. Trong tam giác ABC, nếu A<C, thì wa > wc .
Chứng minh. Ta có
w2a =

4abp( p − c)
4bcp( p − a)
, w2c =
.
2
(b + c)

( a + b )2

Suy ra

( a + b )2 ( c + b )2 2
(wa − w2c ) = c(b + c − a)( a + b)2 − a( a + b − c)(b + c)2
2bp
= b3 (c − a) + b2 (c2 − a2 ) + 3abc(c − a) + ca(c2 − a2 ).
Vì AMệnh đề 8.2. Trong mọi tam giác ABC có bất đẳng thức
w a wb + wb wc + wc w a ≤ r a rb + rb r c + r c r a .
Dấu đẳng thức xảy ra khi và chi khi tam giác ABC đều.

15

(8.1)


Hội thảo Khoa học, Sầm Sơn 28-28/09/2019
Chứng minh. Ta có
wa ≤

p ( p − a ), w b ≤
w a wb ≤ p

p ( p − b ), w c ≤

p ( p − c ).

sc

, ...
2

( p − a)( p − b) ≤

Suy ra
w a w b + w b w c + w c w a ≤ p2 .
Mặt khác, từ các công thức
ra =

S
S
S
, r =
, rc =
p−a b
p−b
p−c

suy ra
r a r b + r b r c + r c r a = p2 .
Từ đó ta có điều phải chứng minh.
Mệnh đề 8.3. Trong mọi tam giác ABC có bất đẳng thức
r a w a + r b w b + r c w c ≤ p2 .

(8.2)

Dấu đẳng thức xảy ra khi và chỉ khi tam giác ABC là đều.
Chứng minh. Từ các công thức
ra =



p( p − b)( p − c)
2 bc
, wa =
p−a
b+c

suy ra
ra wa ≤ p

( p − b)( p − c) ≤

p( p − a)
sa
.
2

Vậy ta có
a+b+c
= p2 .
2

r a w a + rb wb + rc wc ≤ p
Mệnh đề được chứng minh.

Mệnh đề 8.4. Trong mọi tam giác ABC có bất đẳng thức
ra
wa


λ

rb
wb

+

λ

+

rc
wc

λ

≥ 3, λ > 0.

(8.3)

Dấu đẳng thức xảy ra khi và chỉ khi tam giác ABC đều.
Chứng minh. Ta có
ra
wa

λ

( p − b)λ ( p − c)λ
, ...
( p − a)λ




Từ đó suy ra
ra
wa



( p − b)λ ( p − c)λ
+
( p − a)λ

λ

+

rb
wb

λ

+

rc
wc

( p − a)λ ( p − c)λ
+
( p − b)λ


λ

( p − b)λ ( p − a)λ
≥3
( p − c)λ

trên cơ sở của bất đẳng thức AM=GM. Rõ ràng là dấu đẳng thức xảy ra khi và chi khi
a=b=c.

16


Hội thảo Khoa học, Sầm Sơn 28-28/09/2019
Mệnh đề 8.5. Trong mọi tam giác ABC có bất đẳng thức

w2a + w2b + w2c ≥ 3 3S..

(8.4)

Dấu đẳng thức xảy ra khi và chỉ khi tam giác ABC đều.
Chứng minh. Vì
w2a = bc −

bc
1
a2 bc
,
≤ ,
2

2
(b + c)
(b + c)
4

nên

1

∑ w2a ≥ ∑ ab − 4 ∑ a2
Theo các bất đẳng thức Hadwiger-Finsler và Gordon [1]( sẽ được chúng minh dưới dạng
các bồ đề dưới đây)

∑ ab ≥ 4
ta có

1



3S,

∑ w2a ≥ 2 ∑ ab +

∑ a2 ≤ 2 ∑ ab − 4






3S,




3S ≥ 2 3S + 3S = 3 3S.

Vậy bất dẳng thức (8.4) được chúng minh.
Bổ đề 8.1 (Bất đẳng thức Hadwiger-Finsler 1, 1939). Trong mọi tam giác ABC có bất đẳng
thức

a2 + b2 + c2 ≥ ( a − b)2 + (b − c)2 + (c − a)2 + 4 3S.
(8.5)
Chứng minh. Đặt
x = p − a, y = p − b, z = p − c.
Dễ dàng thấy rằng
p = x + y + z, a2 − (b − c)2 = 4( p − b)( p − c) = 4yz,
b2 − ( a − c)2 = 4( p − a)( p − c) = 4xz, c2 − ( a − b)2 = 4( p − a)( p − b) = 4xy.
Do đó bất đẳng thức (8.5) tương đương với bất đẳng thức
4( xy + yz + zx ) ≥ 4

3( x + y + z) xyz.

Bình phương hai vế bất đẳng thức trên ta được

( xy + yz + zx )2 ≥ 3( x + y + z) xyz,
hay

( xy)2 + (yz)2 + (zx )2 ≥ ( xy)(yz) + (yz)(zx ) + (zx )( xy).
Bất đẳng thức cuối cùng hiển nhiên đúng. Dấu đẳng thức xảy ra khi và chỉ khi

x = y = z ⇔ p − a = p − b = p − c ⇔ a = b = c.
Bổ đề được chứng minh.

17


Hội thảo Khoa học, Sầm Sơn 28-28/09/2019
Bổ đề 8.2 (Bất đẳng thức Gordon 1966). Trong mọi tam giác ABC có bất đẳng thức

ab + bc + ca ≥ 4 3S.

(8.6)

Chứng minh. Bất đẳng thức


a2 + b2 + c2 ≥ 4 3S + ( a − b)2 + (b − c)2 + (c − a)2
tương đương với


ab + bc + ca ≥ 4 3S + ( a2 + b2 + c2 − ab − bc − ca).

Vì a2 + b2 + c2 − ab − bc − ca ≥ 0 nên ab + bc + ca ≥ 4 3S. Dấu đẳng thức xảy ra khi và
chỉ khi a=b=c.

9 Bất đẳng thức mới liên quan đến đường phân giác
và đường trung tuyến
9.1

Giới thiệu


Cho tam giác ABC, ta ký hiệu A, B, C là các góc của tam giác, a, b, c là độ dài các cạnh
tương ứng, ký hiệu wa , wb , wc và m a , mb , mc tướng ứng là các đường phân giác góc A, B, C
và các đường trung tuyến. Gọi R, r và p tương ứng là bán kính đường trịn ngoại tiếp,
đường trịn nội tiếp, và nửa chu vi của tam giác, ký hiệu S là diện tích của tam giác ABC.
Các đường phân giác và đường trung tuyến của tam giác có nhiều tính chất thú vị.
Nói riêng, bất đẳng thức về đường phân và đường trung tuyến là chủ đề rất hấp dẫn và
đóng vai trị quan trọng trong nghiên cứu hình học. Một lượng lớn các kết quả liên quan
có thể xem trong các chuyên khảo nổi tiếng [1, 5].
Trong [5] ( tr. 220) trình bày các bất đẳng thức sau
p2 ≤

∑ ma wa ≤ 3(2R2 + r2 ).

(9.1)

Đây là bất đẳng thức Janous mở rộng (theo [7]).
Trong [5]( tr. 219) cũng trình bày bất đẳng thức
ma

∑ wa



13
r

.
4
2R


(9.2)

Mục tiêu chính của mục này là đưa ra một vài bất đẳng thức hình học liên quan đến
đường phân giác và đường trung tuyến của tam giác, vận dụng những bất đẳng thức đó,
có thể cải tiến các bất đẳng thức (9.1) và (9.2).

18


Hội thảo Khoa học, Sầm Sơn 28-28/09/2019

9.2

Một số bổ đề

Để chứng minh kết quả chính của chúng tơi, chúng tơi sẽ sử dụng các kết quả sau.
Bổ đề 9.1. Trong tam giác ABC, ta có
m2a w2a = p2 ( p − a)2 +

S2
( b − c )2 .
( b + c )2

(9.3)

Chứng minh. Theo công thức đường trung tuyến và đường phân giác của tam giác ABC
2 bcp( p − a)
1
: m2a = (2b2 + 2c2 − a2 ) và wa =

, ta dễ dàng tính được
4
b+c
1
m2a = p( p − a) + (b − c)2 ,
4

w2a = p( p − a) −

p( p − a)
( b − c )2 .
( b + c )2

(9.4)

(9.5)

Từ (9.4) và (9.5), ta có
1
m2a w2a = p( p − a) + (b − c)2
4

p( p − a) −

( p( p − a))
( b − c )2
( b + c )2

1 p( p − a)(b − c)4 1
p2 ( p − a )2

2
+
( b − c )2
p
(
p

a
)(
b

c
)

4
( b + c )2
4
( b + c )2
p( p − a)(b − c)2
[(b + c)2 − 4p( p − a) − (b − c)2 ]
p2 ( p − a )2
4( b + c )2
p( p − a)(b − c)2
p2 ( p − a )2
[(b + c)2 − 4(b + c + a)(b + c − a) − (b − c)2 ]
4( b + c )2
p( p − a)(b − c)2 2
p2 ( p − a )2
[ a − ( b − c )2 ]
4( b + c )2

p( p − a)(b − c)2
p2 ( p − a )2
4( p − b)( p − c)
4( b + c )2
p( p − a)( p − b)( p − c)
p2 ( p − a )2
( b − c )2
( b + c )2
S2
p2 ( p − a )2 +
( b − c )2 .
( b + c )2

= p2 ( p − a )2 −
=
=
=
=
=
=

Bổ đề 9.2. Trong

ABC, ta có

( b + c )2
p2 + 10Rr + r2
=
.
∑ bc

2Rr
Chứng minh. Ta có

( b + c )2
1
1
∑ bc = abc ∑ a(b + c)2 = abc ∑ a(2p − a)2
1
=
(4p2 ∑ a − 4p ∑ a2 + ∑ a3 ).
abc

19

(9.6)


Hội thảo Khoa học, Sầm Sơn 28-28/09/2019
Dựa theo các đẳng thức trong [5] (trang 52), ta có

∑ a2 = 2( p2 − 4Rr − r2 ),
∑ a3 = 2p( p2 − 6Rr − 3r2 ),
∑ a = 2p,
abc = 4Rrp,
nên ta thu được (9.6).

9.3

Kết quả chính


Định lý 9.1. Trong tam giác ABC, ta có
p( p − a) ≤ m a wa ≤ p( p − a) +

S2
( b − c )2 .
2p( p − a)(b + c)2

(9.7)

Dấu bằng xảy ra khi và chỉ khi b = c.
Chứng minh. Theo Bổ đề 9.1 ta dễ dàng tính được
m2a w2a ≥ p2 ( p − a)2 ,
từ đó ta thu được vế trái của (9.7).
Mặt khác, theo Bổ đề 9.1, ta có
m2a w2a = p2 ( p − a)2 +

S2
( b − c )2
( b + c )2
S2 ( b − c ) 2
2p( p − a)(b + c)2

2

S2 ( b − c ) 2
≤ p( p − a) +
2p( p − a)(b + c)2

2


= p( p − a) +



S4 ( b − c ) 4
,
4p2 ( p − a)2 (b + c)4

.

Suy ra điều phải chứng minh.
Hệ quả 9.1. Trong tam giác ABC, ta có
1
m a w a ≤ p ( p − a ) + ( b − c )2 .
8

(9.8)

Chứng minh. Theo Định lý 9.1, ta chỉ cần chứng minh
S2
1
<
2p( p − a)(b + c)2
8
hay tương đương với
4S2 − p( p − a)(b + c)2 < 0

⇔ p( p − a)[4( p − b)( p − c) − (b + c)2 ] < 0
⇔ p( p − a)[ a2 − (b − c)2 − (b + c)] < 0
⇔ p( p − a)[( a + b + c)( a − b − c) − (b − c)2 ] < 0.

Vì a < b + c nên (9.9) đúng.

20

(9.9)


Hội thảo Khoa học, Sầm Sơn 28-28/09/2019
Dựa theo Hệ quả 9.1, ta có thể thu được kết quả sau.
Hệ quả 9.2. Trong tam giác ABC, ta có
1

∑ m a w a ≤ p2 + 8

( a − b )2 + ( b − c )2 + ( c − a )2 .

(9.10)

Dấu bằng xảy ra khi và chỉ khi tam giác ABC là tam giác đều.
Nhận xét 9.1. Sử dụng các đẳng thức

∑ a2 = 2( p2 − 4Rr − r2 ), ∑ bc = p2 + 4Rr + r2 ,
thì (9.10) tương đương với
3

5

∑ ma wa ≤ 4 p2 − 3Rr − 4 r2 .

(9.11)


Ta chú ý (9.11) mạnh hơn vế phải của (9.1).
Nhận xét 9.2. Xiao-Guang Chu chứng minh được rằng bất đẳng thức (theo [7]):

∑ ma wa ≤ p2 + 2Rr − 4r2 .

(9.12)

(9.12) cũng mạnh hơn vế phải của (9.1), nhưng ta không thể so sánh (9.11) và (9.12) với
nhau.
Sử dụng bất đẳng thức Gerretsen[1, tr. 50]:
p2 ≤ 4R2 + 4Rr + 3r2
và (9.11), ta có Hệ quả sau đây
Hệ quả 9.3.

∑ ma wa ≤ 5R2 + 2Rr + 3r2 .

Theo Hệ quả 9.1, ta thu được
Hệ quả 9.4. Trong tam giác ABC, ta có
1

∑ ama wa ≤ 4 p( p2 + 18Rr + 9r2 ).

(9.13)

Dấu bằng xảy ra khi và chỉ khi tam giác ABC là tam giác đều.
Dựa vào Hệ quả 9.1 và một vài đẳng thức đã biết, ta thu được
Hệ quả 9.5. Trong tam giác ABC, ta có
1


∑(b + c)ma wa ≤ 2 p(5p2 − 22Rr − 7r2 ).
Dấu bằng xảy ra khi và chỉ khi

(9.14)

ABC là tam giác đều.

Định lý 9.2.



ma
p2 + 10Rr + r2

.
wa
8Rr

Dấu bằng xảy ra khi và chỉ khi tam giác ABC là tam giác đều.

21

(9.15)


Hội thảo Khoa học, Sầm Sơn 28-28/09/2019
Chứng minh. Theo vế trái của (9.7) và Bổ đề 9.2, ta có
ma

∑ wa


≥∑

ma wa
w2a
p( p − a)
=
w2a

=∑

( b + c )2
p2 + 10Rr + r2
=
.
4bc
8Rr

=∑



p( p − a)(b + c)2
4bcp( p0a)

Nhận xét 9.3. Từ bất đẳng thức Gerretsen [1, tr. 50]:
p2 ≥ 16Rr − 5r2 ,
ta có thể thu được (9.2) dễ dàng từ Định lý 9.2, nên (9.15) mạnh hơn (9.2).
Hệ quả 9.6.
ma


∑ wa



5
8p2
+
.
3 81Rr

(9.16)

Dấu bằng xảy ra khi và chỉ khi tam giác ABC là tam giác đều.
Chứng minh. Theo Định lý 9.2, ta chỉ cần chỉ cần chứng minh
5
8p2
p2 + 10Rr + r2
≥ +
,
8Rr
3 81Rr

(9.17)

bất đẳng thức (9.17) tương đương với
17p2 ≥ 270Rr − 81r2 .

(9.18)


Theo bất đẳng thức Gerretsen
p2 ≥ 16Rr − 5r2 ,
và bất đẳng thức Euler [1, tr. 48]
R ≥ 2r,
ta kết luận bất đẳng thức (9.18) đúng.
Thật ra, (9.16) từng là một giả thiết được đề xuất bởi Hua-yan Yin (theo [7]).
Định lý 9.3. Trong tam giác ABC, ta có


ma
3 3 3(2 − 3)
( b − c )2

+
∑ a
∑ bc .
2
2

(9.19)

Dấu bằng xảy ra khi và chỉ khi tam giác ABC là tam giác đều.
Chứng minh. (9.19) tương đương với


1
3 3 3(2 − 3) 1
bcm a ≥
+
a ( b − c )2

abc ∑
2
2 √abc ∑

3(2 − 3)
⇔ ∑ bcm a ≥ 6 3Rrp +
∑ a ( b − c )2 .
2

22

(9.20)


Hội thảo Khoa học, Sầm Sơn 28-28/09/2019
Dựa đẳng thức đã biết

∑ a(b − c)2 = 2p( p2 − 14Rr + r2 ),
ta thu được

∑ bcma ≥ 3p[(2 −





3) p2 + (16 3 − 28) Rr + (2 − 3)r2 ].

(9.21)


Đẳng thức và bất đẳng thức sau được đưa ra trong [5, tr. 242, tr. 244]:

∑ b2 c2 m2a = p6 − 12Rrp4 + r2 p4 + r2 p2 (12R2 + 8Rr − r2 ) − r3 (4R + r)3 .
abc ∑ amb mc ≥ 4Rrp2 ( p2 − 7Rr + 5r2 ).

(9.22)
(9.23)

Dựa vào (9.22) và (9.23), ta có

(∑ bcm a )2 ≥ p6 − p4 (4Rr − r2 ) + p2 (−44R2 r2 + 48Rr3 − r4 ) − r3 (4R + r )3 .

(9.24)

Để chứng minh (9.19), dựa vào (9.24), ta chỉ cần chỉ ra
p6 − p4 (4Rr − r2 ) + p2 (−44R2 r2 + 48Rr3 − r4 ) − r3 (4R + r )3



2
≥ 9p2 (2 − 3)2 + (16 3 − 28) Rr + (2 − 3)r2



⇔ (36 3 − 62) p6 + p4 (1868 − 1080 3) Rr + (72 3 − 125)r2



+ p2 (8064 3 − 14012) R2 r2 + (1920 − 1080 3) Rr3 + (36 3 − 64)r4


− r3 (4R + r )3 ≥ 0

⇔ (36 3 − 62) p4

√ r3 2

p
+ (876 − 504 3) Rr + (123 − 72 3)r2 + (124 − 72 3)
R



r5
+ 4R2 r2 + (384 − 216 3) Rr3 + (180 3 − 324)r4 + (432 3 − 742)
R
3

r6
2r
+ (144 3 − 248)
· p2 − 16Rr + 4r2 +
R
R
4


r
+ 3 ( R − 2r ) · {(6080 − 3456 3) R4 + (5420 − 3168 3) R3 r
R √



+ (288 3 − 505) R2 r2 + (792 3 − 1362) Rr3 + (144 3 − 248)r4 } ≥ 0.
2r3
≥ 0 (xem [7]), R ≥ 2r và các hệ số trong dấu ngoặc không âm,
R
ta suy ra điều phải chứng minh.

Vì p2 − 16Rr + 4r2 +

10 Bất đẳng thức mới liên quan đến đường phân giác
và điểm bên trong tam giác
Nội dung của mục này được hình thành trên cơ tài liệu [8].

23


Hội thảo Khoa học, Sầm Sơn 28-28/09/2019

10.1

Giới thiệu

Như thường lệ, chúng ta xét tam giác ABC với độ dài các cạnh là a, b, c, bán kính
đường trịn ngoại nội tiếp là R và r tương ứng. Ký hiệu m a , mb , mc là đọ dài các đường
phân giác. Giả sử P là điểm bên trong tam giác ABC. Ký hiệu R a , Rb , Rc là bán kính đường
trịn ngoại tiếp của các tam giác PBC, PCA, PAB tương ứng.
Chứng ta sẽ chứng minh Định lý sau đây.
Định lý 10.1. Với mọi điểm trong P và mọi số dương x, y, z, ta có bất đẳng thức
zw
xw

yw
√ a +√ b +√ c ≤
Rb Rc
R a Rb
Rc R a

2+

r yz zx xy
+
+
.
2R x
y
z

(10.1)

Đẳng thức xảy ra khi và chỉ khi tam giác ABC là đều, điểm P là tâm của tam giác và x=y=z.
Chúng ta sẽ cần đến các bổ đề sau đây.
Bổ đề 10.1 (Klamkin). Có bất đẳng thức
λ21 + λ22 + λ23 ≥ (−1)n+1 {2λ2 λ3 cos nA + 2λ3 λ1 cos nB + 2λ1 λ2 cos nC },

(10.2)

trong đó λ1 , λ2 , λ3 là các số thực tùy ý, n là số nguyên, và A, B, C là các góc của tam giác bất kỳ.
Dấu bằng xảy ra khi và chỉ khi
λ2
λ3
λ1

=
=
.
sin nA
sin nB
sin nC

(10.3)

Chứng minh. Ta rút ra ngay lập tức bất đẳng thức (10.2) bằng cách viết nó trong dạng
tương đương sau

{λ1 + (−1)n (λ2 cos nC + λ3 cos nB)}2 + {λ2 sin nC − λ3 sin nB}2 ≥ 0,
n

2

2

{λ2 + (−1) (λ1 cos nC + λ3 cos nA)} + {λ1 sin nC − λ3 sin nA} ≥ 0.

(10.4)
(10.5)

Bổ đề 10.2 (Kooi). Với các số thực λ1 , λ2 , λ3 , λ1 + λ2 + λ3 = 0, thì

( λ1 + λ2 + λ3 )2 R2 ≥ λ2 λ3 a2 + λ1 λ3 b2 + λ1 λ2 c2 .

(10.6)


Dấu đẳng thức xảy ra khi và chỉ khi
λ1
λ2
λ
= 2 2
= 2 2 12
.
a2 ( b2 + c2 − a2 )
b ( c + a2 − b2 )
c ( a + b − c2 )

(10.7)

Chứng minh. Xét bất đẳng thức (10.2) với n=2. Sử dụng công thức cos 2x = 1 − 2 sin2 x
dễ dàng đưa bất đẳng thức (10.2) về dạng

(λ1 + λ2 + λ3 )2 ≥ λ2 λ3 4 sin2 A + λ3 λ1 4 sin2 B + λ1 λ2 4 sin2 C.

(10.8)

Áp dụng Định lý hàm số sine, ta có
4 sin2 A =

a2
b2
c2
2
2
,
4

sin
B
=
,
4
sin
C
=
.
R2
R2
R2

24

(10.9)


Hội thảo Khoa học, Sầm Sơn 28-28/09/2019
Từ (10.9) và (10.8) ta có (10.6). Tiếp theo, vận dụng định lý hàm số sine và cosine, ta có
a2 ( b2 + c2 − a2 )
,
2Rabc
b2 (c2 + a2 − ab2 )
sin 2B = 2 sin B cos B =
,
2Rabc
c2 ( a2 + b2 − c2 )
sin 2C = 2 sin C cos C =
.

2Rabc

(10.10)

sin 2A = 2 sin A cos A =

(10.11)
(10.12)

Từ (10.10)-(10.12) và (10.3) suy (10.7). Bổ đề được chứng minh.
Bổ đề 10.3. Với các số không âm x, y, z có bất đẳng thức
x2 sin2 A + y2 sin2 B + z2 sin2 C ≤

1 yz zx xy
+
+
4 x
y
z

2

.

(10.13)

Đẳng thức xảy ra khi và chỉ khi
x 2 : y2 : x 2 =

a2 ( b2


1
1
1
: 2 2
: 2 2
.
2
2
2
2
+ c − a ) b ( c + a − b ) c ( a + b2 − c2 )

Chứng minh. Áp dụng Bổ đề 10.2 với
yz
zx
xy
λ1 =
, λ2 =
, λ3 =
,
x
y
z
thì khẳng định (10.13) được suy ra từ định lý hàm số sine
a = 2R sin A, b = 2R sin B, c = 2R sin C.

10.2

Chứng minh Định lý 10.1


Đối với đường phân giác của góc A ta có
wa =



2bc
A
cos .
b+c
2

A
, đẳng thức xảy ra khi và chỉ khi b=c.
2
Giả sử ∠ BPC = α, ∠CPA = β, ∠ APB = γ. Rõ ràng là 0 < α, β, γ < π và α + β +
γ = 2π. Theo Định lý hàm số sine ta có

Rõ ràng là wa ≤

bc cos

b = 2Bb sin β, c = 2Rc sin γ.
Ta có



wa

Rb Rc


bc
A
cos
Rb Rc
2

A
2
A
≤ (sin β + sin γ) cos
2
β+γ
β−γ
A
= 2 sin
cos
cos
2
2
2
α
A
≤ 2 sin cos .
2
2

= 2 sin β sin γ cos

25



×